2014 dxdy logo

Научный форум dxdy

Математика, Физика, Computer Science, Machine Learning, LaTeX, Механика и Техника, Химия,
Биология и Медицина, Экономика и Финансовая Математика, Гуманитарные науки




Начать новую тему Ответить на тему На страницу Пред.  1 ... 9, 10, 11, 12, 13, 14, 15 ... 27  След.
 
 
Сообщение14.07.2007, 15:36 


14/02/06
285
Два последних моих поста неверны - невнимательность - отвечал не на тот вопрос.

Прошу прощения, у меня полночь.

Теперь вижу ,что контрпример arcady работает.
Приношу Вам свои извинения за необоснованное ехидство.

 Профиль  
                  
 
 
Сообщение14.07.2007, 17:14 
Заслуженный участник


26/06/07
1929
Tel-aviv
Всё нормально, sergey1. Вы же были уверены в своей правоте.
А не ошибается тот, кто ничего не делает. Всё нормально!

 Профиль  
                  
 
 
Сообщение15.07.2007, 21:53 


15/03/07
128
Господин Артомонов, Вы рассуждаете, похоже, неверно.
В самом деле, Вы говорите, что $AB >= 9$
Нам необходимо показать, что $A >= \frac{1}{2}$.
Тогда нам необходимо (или достаточно - не знаю) показать, что
$B <= 18$ ( при этом $A >= \frac{9}{B} >= \frac{1}{2}$ ).
Но нер-во $B < 18$ не имеет места (более того, по-моему $B >= 18$ , причем
выражение $B$ принимает min, в силу наших ограничений, при $a=b=c=1$).

 Профиль  
                  
 
 
Сообщение15.07.2007, 22:02 


10/03/07
59
Казань
Можно несколько упростить задачу, если сделать её симметричной. Перепишем неравенство в виде:
$2[a(c^2+5)(a^2+5) + b(b^2+5)(a^2+5)+c(c^2+5)(a^2+5)]\geqslant(c^2+5) (a^2+5) (b^2+5) $
Применяя теорему К.-Б., усилим неравенство, заменяя левую часть на:
$2\sqrt { a^2+b^2+c^2} \sqrt{(c^2+5)^2 (a^2+5)^2+(b^2+5)^2(a^2+5)^2+ (c^2+5)^2(a^2+5)^2}$.
Применяя ко второму корню неравенство между средними, еще раз усилим левую часть:
$2\sqrt{3}\sqrt { a^2+b^2+c^2}(c^2+5)^{2/3} (a^2+5)^ {2/3}(b^2+5)^{2/3} \geqslant(c^2+5) (a^2+5) (b^2+5) $, или
$\frac{(c^2+5)^2 (a^2+5)^2 (b^2+5)^2}{ (a^2+b^2+c^2)^3} \leqslant 12^3$.
Судя по машинному графику, это усиленное неравенство также справедливо в заданной области. Хотя оно тоже не слишком простое, и доказать его я пока не успел.

Могу предложить в качестве побочного продукта занятные образцы, которые, однако, не принесли пользы для решения задачи.
$ab^2 + bc^2+ca^2\leqslant a^3+b^3+c^3$ для произвольных $a,b,c$, а также:
$a^2b^3 +b^2c^3 + c^2a^3\geqslant \frac{1}{3} (ab +bc +cd)^2$ для $\sum=3$.

 Профиль  
                  
 
 
Сообщение15.07.2007, 23:00 
Заслуженный участник


26/06/07
1929
Tel-aviv
Скорцонер писал(а):
Можно несколько упростить задачу, если сделать её симметричной. Перепишем неравенство в виде:
$2[a(c^2+5)(a^2+5) + b(b^2+5)(a^2+5)+c(c^2+5)(a^2+5)]\geqslant(c^2+5) (a^2+5) (b^2+5) $
Применяя теорему К.-Б., усилим неравенство, заменяя левую часть на:
$2\sqrt { a^2+b^2+c^2} \sqrt{(c^2+5)^2 (a^2+5)^2+(b^2+5)^2(a^2+5)^2+ (c^2+5)^2(a^2+5)^2}$.

Но в силу К-Б получаем:
$2\sqrt { a^2+b^2+c^2} \sqrt{(c^2+5)^2 (a^2+5)^2+(b^2+5)^2(a^2+5)^2+ (c^2+5)^2(a^2+5)^2}\geq$
$\geq2[a(c^2+5)(a^2+5) + b(b^2+5)(a^2+5)+c(c^2+5)(a^2+5)] .$
Согласно вашей терминологии, вы не усилили, а ослабили исходное неравенство.
То-бишь уже ничего нет. :wink:

Добавлено спустя 24 минуты 3 секунды:

Скорцонер писал(а):

Могу предложить в качестве побочного продукта занятные образцы, которые, однако, не принесли пользы для решения задачи.
$ab^2 + bc^2+ca^2\leqslant a^3+b^3+c^3$ для произвольных $a,b,c$, а также:
$a^2b^3 +b^2c^3 + c^2a^3\geqslant \frac{1}{3} (ab +bc +cd)^2$ для $\sum=3$.

Думаю, вы имеете в виду, что речь идёт о неотрицательных числах. :wink:
Первое можно доказать, например, так:
$ab^2 + bc^2+ca^2\leq a^3+b^3+c^3\Leftrightarrow\sum_{cyc}(a^3+2b^3-3ab^2)\geq0,$ что очевидно согласно AM-GM.
Второе - что-то интересное. Щас попробую.
Вы, конечно, имели в виду вот это:$a^2b^3 +b^2c^3 + c^2a^3\geqslant \frac{1}{3} (ab +bc +ca)^2$

Добавлено спустя 5 минут 40 секунд:

Да нет! Оно очевидно неверно при c=0. :wink:

 Профиль  
                  
 
 
Сообщение16.07.2007, 11:54 
Заслуженный участник
Аватара пользователя


07/03/06
1898
Москва
Pyphagor писал(а):
Господин Артомонов, Вы рассуждаете, похоже, неверно.
В самом деле, Вы говорите, что $AB >= 9$
Нам необходимо показать, что $A >= \frac{1}{2}$.
Тогда нам необходимо (или достаточно - не знаю) показать, что
$B <= 18$ ( при этом $A >= \frac{9}{B} >= \frac{1}{2}$ ).
Но нер-во $B < 18$ не имеет места (более того, по-моему $B >= 18$ , причем
выражение $B$ принимает min, в силу наших ограничений, при $a=b=c=1$).

Я рассуждал так.
Имеем $\frac{a}{b^2+5}+\frac{b}{c^2+5}+\frac{c}{a^2+5}=\frac{3}{5}+\frac{a^2c(a^2-5)+b^2a(b^2-5)+c^2b(c^2-5)}{5^3}+$ $\frac{a^6c(a^2-5)+b^6a(b^2-5)+c^6b(c^2-5)}{5^5}+...$. Ряд абсолютно сходится при $a,b,c<\sqrt{5}$, тогда каждый член правой части $\frac{a^2c(a^2-5)+b^2a(b^2-5)+c^2b(c^2-5)}{5^3}+$ $\frac{a^6c(a^2-5)+b^6a(b^2-5)+c^6b(c^2-5)}{5^5}+...$ отрицателен, и мы последовательно приближаемся к значению искомого выражения. Известно, что первые члены ряда играют определяющую роль в окончательном значении выражения. Поэтому, если мы хотим спуститься к $\frac 1 2$ или ниже, мы должны найти такие $$a,b,c$, при которых эти первые члены ряда принимают максимально отрицательное значение. Я учитываю два члена ряда, которые регулируют первый и второй знак. Остальные члены регулируют последующие знаки и как бы мы их не изменяли, если первые члены не минимальны, мы не спустимся ниже того, когда они минимальны, кроме возможно некоторой малой окрестности около точки, в которой первые члены минимальны. Поэтому решаем оптимизационную задачу $a^2c(a^2-5)+b^2a(b^2-5)+c^2b(c^2-5)\to\min\text{ при } a+b+c=3$ и находим $a=b=c=1$. Таким образом, мы доказали, что искомое неравенство будет верно при $a,b,c<\sqrt{5}$, кроме возможно некоторой окрестности точки $(1,1,1)$
Но, во-первых, это не решает полностью задачу, во-вторых, строго аналитически решить указанную оптимизационную задачу сложно (если и возможно), в третьих, такое решение сильно не по душе arqady, в четвертых, оно не нравится и мне.

 Профиль  
                  
 
 Re: Нестандартные задачи
Сообщение17.07.2007, 15:53 
Заслуженный участник


05/09/05
515
Украина, Киев
В этой теме одна задача рождает другую, а другая третью, так что я и не понял: решена ли одна из исходных задач?

student писал(а):

3)Докажите для положителные действительные числа $a,\ b,\ c$ справедливо
$\frac{a}{b+2c}+\frac{b}{c+2a}+\frac{c}{a+2b}\geq 1$



------
"Джентельмены, а поезд уже ушел?"
из к-ма "Человек с бульвара Капуцинов."

 Профиль  
                  
 
 Re: Нестандартные задачи
Сообщение17.07.2007, 17:42 
Заслуженный участник


26/06/07
1929
Tel-aviv
Macavity писал(а):
В этой теме одна задача рождает другую, а другая третью, так что я и не понял: решена ли одна из исходных задач?

student писал(а):

3)Докажите для положителные действительные числа $a,\ b,\ c$ справедливо
$\frac{a}{b+2c}+\frac{b}{c+2a}+\frac{c}{a+2b}\geq 1$



Эта задача очень лёгая и может быть решена многими способами, например, так:
$\frac{a}{b+2c}+\frac{b}{c+2a}+\frac{c}{a+2b}=\sum_{cyc}\frac{a^2}{ab+2ac}\geq\frac{(a+b+c)^2}{3(ab+ac+bc)}\geq1.$

 Профиль  
                  
 
 Re: Нестандартные задачи
Сообщение17.07.2007, 18:42 
Заслуженный участник


05/09/05
515
Украина, Киев
arqady писал(а):
Macavity писал(а):
В этой теме одна задача рождает другую, а другая третью, так что я и не понял: решена ли одна из исходных задач?

student писал(а):

3)Докажите для положителные действительные числа $a,\ b,\ c$ справедливо
$\frac{a}{b+2c}+\frac{b}{c+2a}+\frac{c}{a+2b}\geq 1$



Эта задача очень лёгая и может быть решена многими способами, например, так:
$\frac{a}{b+2c}+\frac{b}{c+2a}+\frac{c}{a+2b}=\sum_{cyc}\frac{a^2}{ab+2ac}\geq\frac{(a+b+c)^2}{3(ab+ac+bc)}\geq1.$


А может быть решение распространено на n-мерный случай?
И верно ли неравенство для n>3 (для n=2 и n=3 оно очевидно верно).

\sum_{i}\frac{a_i}{2a_{i+1mod(n)}+\sum_{j \neq i \wedge j \neq i+1mod(n) }{ a_j}} \geq1

Как-то так...

 Профиль  
                  
 
 
Сообщение17.07.2007, 21:04 
Заслуженный участник


26/06/07
1929
Tel-aviv
Macavity
Это верно по той же причине. Там всё сводится вот к такой радости: $$\sum_{cyc}(a_{i+1}-a_i)^2\geq0.$$
А вот это неравенство
$$\sum_{cyc}\frac{a_1}{a_2+2a_3+...+(n-1)a_n}\geq\frac{2}{n-1}$$
для положтельных $a_i$ и для натурального $n\geq2$ сводится к $$\sum_{sym}(a_i-a_j)^2\geq0.$$ :wink:

 Профиль  
                  
 
 
Сообщение18.07.2007, 13:23 
Заслуженный участник
Аватара пользователя


07/03/06
1898
Москва
Похоже верно следующее утверждение:
$\forall a,b,c\geqslant 0,a+b+c=3: \frac{a}{b^2+5}+\frac{b}{c^2+5}+\frac{c}{a^2+5}\geqslant \frac{2a}{b+11}+\frac{2b}{c+11}+\frac{2c}{a+11}$.
Тогда можно будет доказать неравенство:
$\frac{a}{b+11}+\frac{b}{c+11}+\frac{c}{a+11}\geqslant \frac{1}{4}$, а для него и метод неопределенных множителей Лагранжа не представляется таким уж неподъемным или разложением в ряд, как я уже показывал.
Вообще, при ограничениях на $a,b,c$ можно и дальше опускаться, но уже до тривиального $\frac{2a}{b+11}+\frac{2b}{c+11}+\frac{2c}{a+11}\geqslant \frac{a}{6}+\frac{b}{6}+\frac{c}{6}=\frac{1}{2}$, видимо можно и подниматься.

 Профиль  
                  
 
 
Сообщение18.07.2007, 17:05 
Заслуженный участник


26/06/07
1929
Tel-aviv
Артамонов Ю.Н. писал(а):
Похоже верно следующее утверждение:
$\forall a,b,c\geqslant 0,a+b+c=3: \frac{a}{b^2+5}+\frac{b}{c^2+5}+\frac{c}{a^2+5}\geqslant \frac{2a}{b+11}+\frac{2b}{c+11}+\frac{2c}{a+11}$.

По-моему, оно неверно для $c=0$ и $a=b=1.5$ :wink:

 Профиль  
                  
 
 
Сообщение18.07.2007, 19:15 
Заслуженный участник
Аватара пользователя


07/03/06
1898
Москва
Ну можно улучшать: берем $\frac{ak}{b+6k-1}+\frac{bk}{c+6k-1}+\frac{ck}{a+6k-1}$, где $k$ - некоторое число, например, $k=11$.

 Профиль  
                  
 
 
Сообщение18.07.2007, 21:21 
Заслуженный участник


26/06/07
1929
Tel-aviv
Артамонов Ю.Н.
Так выберите окончательный вариант и приведите полное доказательство.

 Профиль  
                  
 
 
Сообщение19.07.2007, 10:34 
Заслуженный участник
Аватара пользователя


07/03/06
1898
Москва
Приведу схему доказательства:
1. Нужно показать, что $\exists k_0 \forall k\geqslant k_0: \frac{a}{b^2+5}+\frac{b}{c^2+5}+\frac{c}{a^2+5}\geqslant \frac{ak}{b+6k-1}+\frac{bk}{c+6k-1}+\frac{ck}{a+6k-1}$;
2. $\forall k_1<k_2: \frac{ak_1}{b+6k_1-1}+\frac{bk_1}{c+6k_1-1}+\frac{ck_1}{a+6k_1-1}\geqslant \frac{ak_2}{b+6k_2-1}+\frac{bk_2}{c+6k_2-1}+\frac{ck_2}{a+6k_2-1}$.
Тогда очевидно, что $\forall k\geqslant 2: \frac{ak}{b+6k-1}+\frac{bk}{c+6k-1}+\frac{ck}{a+6k-1}\geqslant \frac 1 2 $, т.к. при $k\to\infty$ получим ровно $\frac 1 2$.
Осталось доказать п.1 и п. 2. :)

 Профиль  
                  
Показать сообщения за:  Поле сортировки  
Начать новую тему Ответить на тему  [ Сообщений: 401 ]  На страницу Пред.  1 ... 9, 10, 11, 12, 13, 14, 15 ... 27  След.

Модераторы: Модераторы Математики, Супермодераторы



Кто сейчас на конференции

Сейчас этот форум просматривают: wrest


Вы не можете начинать темы
Вы не можете отвечать на сообщения
Вы не можете редактировать свои сообщения
Вы не можете удалять свои сообщения
Вы не можете добавлять вложения

Найти:
Powered by phpBB © 2000, 2002, 2005, 2007 phpBB Group